Proving Orthogonal Projection and Norm using Inner Products

  • Thread starter Thread starter Zoe-b
  • Start date Start date
  • Tags Tags
    Orthogonal
Click For Summary
The discussion focuses on proving that the norm of the orthogonal projection T of a vector v onto the orthogonal complement U of a subspace W is equal to the infimum of the distances from v to points in W. The user initially struggles with the proof, unsure how to demonstrate that the projection minimizes the distance ||v - w|| for w in W. They realize that using the projection theorem and concepts from inner product spaces can help, despite not being familiar with Hilbert spaces. A key insight involves visualizing the relationship between the vectors using a right triangle and applying the Pythagorean theorem. Ultimately, the user successfully completes the proof using inner product notation, gaining confidence in their understanding.
Zoe-b
Messages
91
Reaction score
0

Homework Statement


Let U be the orthogonal complement of a subspace W of a real inner product space V.
Have already shown that T is a projection along a subspace W onto U, and that V is the direct sum of W and U.

The questions now says: show
||T(v)|| = inf (w in W) || v - w ||


Homework Equations


I have some vague notion that in R^3 say, an orthogonal projection can be used to find the shortest distance between a plane and a point. I have absolutely no idea how to prove this using inner products though.



The Attempt at a Solution



if we write T' for the projection along U onto W, then we have:

v = (T + T')(v)

T(v) = v - T'(v)

now T'(v) is in W, but I don't know how to show it is the w that minimises || v - w ||

Any suggestions for resources would also be welcome- this is not in my notes at all and google hasn't been that helpful :P
 
Physics news on Phys.org
There is a theorem called 'projection theorem' which gives you exactly that, but it only works in Hilbert spaces, so I'm not sure if it's general enough.
 
Let w_0=v-T(v), then w is supposed to be the point in W which is closest to v. Can you prove that for each w in W holds that

\|v-w_0\|\leq \|v-w\|

Hint: draw a picture and see if you get something perpendicular. Use Pythagoras theorem.
 
Thanks for the replies- Hilbert spaces aren't on my course yet so I don't think that's the way to go. I'm trying to use the second hint and do it with a diagram but not getting that far- also this is in any real inner product space not necessarily R^n..
 
T(v) is orthogonal to W, right??

So v-w0, w-w0 and v-w forms a right triangle.
 
Yeah done it now, thanks! I guess I did use pythagoras- Just conceptually not that happy with drawing triangles when the elements I'm using aren't necessarily vectors? Anyway, done it using just inner product notation and now happy :)
 
Question: A clock's minute hand has length 4 and its hour hand has length 3. What is the distance between the tips at the moment when it is increasing most rapidly?(Putnam Exam Question) Answer: Making assumption that both the hands moves at constant angular velocities, the answer is ## \sqrt{7} .## But don't you think this assumption is somewhat doubtful and wrong?

Similar threads

Replies
1
Views
1K
  • · Replies 2 ·
Replies
2
Views
1K
Replies
5
Views
2K
Replies
8
Views
2K
  • · Replies 1 ·
Replies
1
Views
1K
  • · Replies 1 ·
Replies
1
Views
1K
  • · Replies 4 ·
Replies
4
Views
2K
  • · Replies 1 ·
Replies
1
Views
2K
  • · Replies 8 ·
Replies
8
Views
2K
  • · Replies 7 ·
Replies
7
Views
2K